Using Fourier Transforms to solve Definite Integrals with Limits 0 to Infinity

Click For Summary
Fourier Transforms can simplify the process of solving definite integrals with limits from 0 to infinity, as opposed to calculating indefinite integrals. The discussion highlights the challenge of integrating functions that are neither even nor odd, particularly when exponential decay is involved. Participants express uncertainty about manipulating the integral, especially with the presence of both even and odd components. The conversation emphasizes the need for a clearer strategy to handle these types of integrals effectively. Understanding the properties of even and odd functions is crucial for applying Fourier Transforms in this context.
yellowhappy
Messages
3
Reaction score
0
1. Using Fourier Transforms to solve Definite Integrals with Limits 0 to Infinity

I'm trying to understand how to use Fourier Transforms to solve Definite Integrals with limits from 0 to Infinity.
I understand how to use Fourier Transforms to solve indefinite integrals, but I believe there is supposed to be a much simpler way to solve these definite integrals without working out the entire indefinite solution.


Homework Equations


X(j0) = Integral from -Infinity to Infinity : x(t)dt
x(0) = Integral from -Infinity to Infinity: X(jw)dw
 
Physics news on Phys.org
Welcome to PF! Here's a hint: Think about even and odd time functions.
 
\displaystyle\int^\infty_0 2te^-^a^t\cos(t)\,dt = ?}

I'm not entirely sure how to manipulate this.. the e^-^a^t makes it so the function is neither even nor odd anymore.. even if I pulled the even part out to rewrite the function so its integrated from -infinity to infinity I'm unsure about what to do with the odd piece.
 
Yikes! I don't know what to do with this either. Sorry I jumped in...
 
marcusl said:
Yikes! I don't know what to do with this either. Sorry I jumped in...

I appreciate the thought! Thanks! :)
 
Question: A clock's minute hand has length 4 and its hour hand has length 3. What is the distance between the tips at the moment when it is increasing most rapidly?(Putnam Exam Question) Answer: Making assumption that both the hands moves at constant angular velocities, the answer is ## \sqrt{7} .## But don't you think this assumption is somewhat doubtful and wrong?

Similar threads

  • · Replies 5 ·
Replies
5
Views
2K
  • · Replies 6 ·
Replies
6
Views
3K
  • · Replies 2 ·
Replies
2
Views
3K
Replies
1
Views
2K
  • · Replies 7 ·
Replies
7
Views
2K
Replies
2
Views
2K
  • · Replies 3 ·
Replies
3
Views
2K
  • · Replies 3 ·
Replies
3
Views
2K
  • · Replies 20 ·
Replies
20
Views
2K
Replies
1
Views
2K